You are on page 1of 41

Land Law Mega Document

Introduction
Legal definition of Land Section 205(1) Para 9 Law of Property Act 1925
Includes the land, minerals below it and the airspace above it. Subject to exceptions eg gas or coal
Also it includes things physically attached to land, such as buildings or trees within the def of land
called corporeal hereditaments, as well as intangible rights such as a right of way called incorporeal
hereditaments.
All land owned by the crown Doctrine of tenure
You do not own the land, you own a bundle of rights and duties in relation of land eg right of
possession.
Personal rights are known as rights in personam and can only be enforced against a specific person.
Proprietary rights are known as rights in rem and are rights which can exist in the land itself.
Proprietary rights are capable of binding a new owner of land.
Estates in Land
Legal estates are a type of proprietary right.
The other type of right would be an equitable estate.
What can be legal?
S1(1) LPA 1925 Only two estates can be legal
1. Freehold estate Fee simple absolute in possession
2. Leasehold estate Term of years absolutely
Fee simple Means it is a perpetual grant, capable of being inherited and therefore can last forever.
Absolute Means there is no risk the estate may end at some point in the future.
In possession Means you have an immediate entitlement to enjoy the full rights of an owner.
Leaseholds
Term of years Lease must be granted for a fixed period of time which is known at the date of the
grant and not for an indefinite period. Established in Lace v Chantler
Absolute seems to have no significance and will not stop it being a legal estate just because it can
be terminated at an earlier date.
A lease which commences in the future is called a reversionary lease.

Leases
Tenant can be called a lessee and the landlord a lessor
A term of years absolute, or leasehold estate, is one of the two legal estates listed in S 1 of Law of
Property Act 1925. Granted for a specified period of time.
A lease can be granted by the owner of the freehold estate granting a lease for a certain term. It can
be of any duration.
The lease carved out of the freehold estate, which would be known as the freehold reversion, and
the tenant has a lease of the property giving them the right to occupy and live there. Can be many
interests in land.
Not all leases are legal estates, for example if it is not completed with a deed, then it cannot be
legal. As S.52 of the Law of Property Act states that a deed is needed to create a legal estate.
A transfer of a lease is called an assignment, once it is created it is possible to sell or 'assign' that
lease to another person who would then become the tenant.
Note, you can also assign the freehold reversion, which is just a transfer of a freehold estate which
is subject to an existing lease.
Instead of assigning the lease as to leave no proprietary right left in the house. You can grant a sublease of the house. This is an entirely new leasehold estate in the house, but it must be for a shorter
term than the lease out of which it is given, if only for one day.
In this instance, the landlord would have no legal relationship with the tenant of the sublease.
Termination of a lease
In a fixed term lease, it will usually terminate at the end of the contractual period. However, there
are ways it can be terminated before expiration. Eg, if the tenant is in breach of a covenant in the
lease, the landlord may use the remedy of forfeiture to end the lease prematurely. Or the lease
document may expressly allow for either party to end the lease early by giving the other party a
notice of their intention to terminate.
A lease may end where the tenant surrenders their lease with the consent of the landlord. Or if there
is a merger of the estates so the lease and the freehold reversion come into single ownership.
Legal interests in land
S 1(2) Law of Property Act 1925 lists 5 legal interests
The most important interests are the easement and the mortgage
Easements are capable of being legal if it is granted 'for a term equivalent to an estate in fee simple
in absolute possession or a term of years absolute.' Therefore, for it to be legal it must be granted to
last for either a fixed term or forever.

Another legal interest under S.1(2) LPA 1925 is the charge by way of a legal mortgage. The bank is
the receiver of the mortgage and is known as the mortgagee and the tenant or buyer who is granting
the mortgage is known as the mortgagor.
Less common legal interests are;
1. A landlords right of re-entry in relation to a lease Solicitors will ensure the lease contains a
re-entry clause and is also known as a forfeiture clause, and the purpose is to allow the
remedy of forfeiture available if the tenant breaches one of the covenants of the lease. Eg, if
tenant promised only to use the property for residential purposes, but instead is running a
business from it, tenant is in breach of the covenant.
2. rentcharges
3. statutory charges
S52 LPA 1925 Correct documentation has to be used in order for a right to be legal. A deed must
be used.
Equitable interests in Land
If it is not a legal estate listed in S.1(1) LPA 1925 or any of the legal interests listed in S.1(2) then,
S.1(3) tells us that everything else is an equitable interest.
Examples
Restrictive covenants selling part of your own estate, for another to build a house, a restrictive
covenant could dictate the terms, to make sure they don't build a block of flats on that land etc. This
would be included in the transfer deed. A restrictive covenant benefits my house and is a burden on
the garden or land sold.
Importantly These are always equitable interests even though they are contained in a deed, as they
are not listed in S.1(2).
Another equitable interest is an estate contract. This a contract to create or transfer a legal estate. To
be an estate contract the document must comply with the correct formalities for a contract dealing
with land.
An option to purchase and the right of pre-emption are also examples of estate contracts.
An option to purchase is a promise in a contract that he will sell the property to a specific person if
they want to buy it in the next 5 years. They don't have to buy it, but if they decide to, he must sell it
to them.
Right of pre-emption is a right of first refusal. If he decides to sell the property, and has contracted
that he will offer to his friend first before the general market, then he must do so. He doesn't have to
sell, but if he decided to, then he must offer it there first.
These are all examples of an estate contract and this is always an equitable interest.
Home rights are another example of an equitable interest, they are conferred by statute Family
Law Act of 1996 as amended by the Civil Partnership Act of 2004. A home right is a personal right

given to a spouse or civil partner and protects their right to occupy the matrimonial home.
If a married couple or civil partners buy a house together and both are named as the legal estate
owner, then neither have home rights, there is no need for them.
Therefore home rights only arise where a married couple or civil partners live in a house together
and only one of them is the legal estate owner. The other may have no proprietary interest in the
property at all or they may have the equitable interest of a beneficiary under a trust of land.
Equity and Trusts of Land
The trust of land is the means by which property may be held by one person, called the trustee for
the benefit of another person, called the beneficiary.
A trustee holds what is called the paper title, as she holds the property for the benefit of the
beneficiary.
Tinsley v Milligan House was purchased by two women and both contributed equally to the
purchase price, but only was put into Tinsley's name. A dispute arose. The court held that while
Tinsley was the legal owner, she held half the property on trust for Milligan as an equitable owner.
Tinsley is the trustee for herself and milligan as beneficiaries.
This is an interest of a beneficiary under a trust of land and it is always equitable as it is not listed in
S1(2) as one of the legal interests.
Formalities -Deeds and Contracts
Under S.52 of the Law of Property Act 1925 a deed is needed to CREATE or transfer a legal estate
or a legal interest. There are exceptions however;
Section 54(2) of LPA 1925 states that it is possible to grant a short lease not exceeding 3 years
without using a deed and nevertheless create a legal estate.
However, three requirements must be met;
1. Short lease must be in possession starts on the day it is granted if not it would be a
reversionary lease.
2. Short lease must be at the best rent This means market value rather than a discounted rent
3. Must be no payment of initial premium or fine, This would be a lump sum made at the start
of the lease, but it is not a deposit, ie the tenant would not get it back.
What is a deed?
It must comply with the requirements of S.1 of the Law of Property (Miscellaneous Provisions) Act
1989 and applies to anything created after 31st july 1990.
Firstly, the document must be clear on the face of it that its a deed. All this needs is for deed to be
contained somewhere in the document.
Secondly, the document must be validly executed as a deed. This means the document must be
signed, witnessed and delivered. Must be signed by a relevant person in the presence of a witness
who then signs the document to attest the signature and is finally delivered as a deed by that person
or someone authorised by him on his behalf.

As long as it complies with these two points, it could be written on toilet roll.
Equitable rights do not need to be made by deed. However, estate contracts and rights that could
have been legal but were not made by deed, do need to be made using a contract to be valid as
equitable rights. This is required by S.53(1)(c) LPA 1925.
To be valid as a contract it must meet the requirements of S.2 of the Law of Property
(Miscellaneous Provisions) Act 1989.
Firstly, it must be made in writing and cannot deal with land orally.
Secondly, it must also be signed by or on behalf of each party to the contract, there is no need for
witnesses.
Finally, the contract must incorporate all the terms to which the parties have expressly agreed into
one document. The terms may be incorporated into the contract either by being included in the text
of the document or by reference to another document.
First stage of buying a house is known as the ' exchange of contracts ' and this is the point that the
buyer and seller are bound to complete the sale by a legally enforceable contract. Up until this
point, neither is bound to complete the sale transaction. At this stage the buyer only has an equitable
interest in the land to be purchased through the estate contracts. The equitable maxim 'equity
regards as done that which ought to be done' would apply here as there is a specifically enforceable
contract the buyer could go to court for the remedy of specific performance if the seller refused to
transfer the legal estate.
After this, the seller holds the property on trust for the buyer until a deed is used to transfer the legal
estate to the buyer. A standard form transfer deed is called the TR1 form.
The Issue of Priority
The issue of priority arises from the fact that multiple rights can exist in land.
One of the major roles of land law is to devise a set of rules which deals with these two issues,
which protects holders of interests in land, but also protects the future buyers of that land.
Two systems of rules exist;
The most important are contained within the Land Registration Act 2002 which replaced the Land
Registration Act 1925. This will govern whether a new owner will be bound by restrictive
covenants, leases, mortgages and easements.
The second system relate to unregistered land This is not covered in the syllabus.

Registered Land System


Registered land describes where there is a registered title held with HM Land Registry. Governed
by the Land Registration Act 2002.
Registered title is a legal estate has been entered onto the Title Register, which is held at HM Land
Registry. England and Wales are divided up into areas and each one has its own District Land
Registry.
It is possible to have two or more registered titles in one piece of land, eg, the freehold estate may
be registered alongside a leasehold estate for the same property. The property can be referred to as
registered land if it has at least one registered title.
It was created to simplify conveyancing the buying and selling of land.
Two important checks when buying a house.
1. Does the seller own the house?
2. Are there any 3rd party rights which burden the land? Ie easements, restrictive covenants?
You can find this out by checking the title register.
How it works;
When an estate is registered it is given an individual register and its own unique title number
allocated by the registrar. Other information is also recorded on the register, for example interests
which benefit or burden the land.
Each title is divided into 3 parts:
1. Property register Identifies the land, usually finding a description of the property and it's
postal address. Will also indicate whether it is a freehold or a leasehold estate. This register
will also include details of any rights which benefit the land and form part of the property.
2. Proprietorship register This identifies the owners of the land, which are called the
registered proprietors. This part will also contain the grade of the title. Finally, this will
contain any Restrictions, these are a particular method entering certain matters onto the
register which limit the power of registered proprietors to deal with their land.
3. Charges register This details any 3rd party rights which burden the land, eg, legal
mortgages, restrictive covenants in freehold land, estate contracts, home rights and
easements.
The LRA has a number of principal aims;
1. Bring all unregistered land in England and Wales onto the register as soon as possible. It
seeks to achieve this aim by increasing the number of trigger events which trigger the need
for compulsory first registration.
2. To ensure that as many 3rd party rights as possible are recorded on the title register. They aim
to do this by reducing the number of overreaching interests.
There are limited instances for the register to be altered. If this occurs compensation may be made
payable by the Land Registry to anyone who suffers a loss by reason of the alteration of the register.

First registration of Title


This occurs where an event triggers the need for registration of an estate that is currently
unregistered.
There is an opportunity under S.3 LRA 2002 to voluntarily register title and provides reduced fees.
Section 4 lists those events which trigger the need for compulsory first registration of title.
Trigger events;
1.
2.
3.
4.
5.

A transfer of a freehold estate triggers the need for compulsory first registration,
The grant of a legal lease of more than 7 years,
The transfer of a legal lease with more than 7 years to run will also trigger it
para (d) states, that the creation of a reversionary lease of any length must be registered.
The grant of a first legal mortgage over a freehold estate or a legal lease of 7 years.

There is more than one way to transfer the property, sale is one way, but so is by way of a gift.
Following a trigger event the relevant party only has 2 months to fill in the forms and register the
estate under S.6(4) LRA 2002, this begins on the date that the relevant event occurs. If this is not
completed, under S.7 the transfer or grant of the legal estate is void.
When applying for first registration, whether compulsory or voluntary registration, the registrar will
investigate the title and check the title documents. On the basis of this check, the registrar will give
the title a grade which indicates how reliable the title is considered to be.
Dealing with Registered Land S.27 LRA 2002
When referring to dealing with registered land, it means the owner creating or transferring an estate
or interest out of their registered land.
S.27 LRA 2002 gives a list of those dealings which must be registered;
1. Every transfer of a registered estate both freehold and leasehold, even if there is less than 7
years remaining.
2. Grant of a lease of more than 7 years
3. If a reversionary lease is created
4. The grant of a legal charge
5. Expressly created easements
S.27 states that if a disposition is required to be completed by registration, it does not operate at law
until the relevant requirements are met. This would mean the legal estate would remain with the
seller and the buyer would only acquire an equitable ownership. The same applies to the rest of
them, they would only acquire an equitable version.
This creates an additional requirement for easements, not only must they be forever or a fixed term,
and comply with s.52 LPA 1925 and be made via deed for a legal interest, they must also be
registered under s.27, otherwise it will only be an equitable easement.

Entries on the Register


Under the LRA 2002, there are two methods of entering an interest onto the title register, namely
the notice and the restriction.
Notices are governed by sections 32 to 39 of the LRA 2002.
S.32(1) defines a notice as 'an entry in the register in respect of the BURDEN of an interest
affecting a registered estate'
This means where a 3rd party interest burdens the land, as opposed to benefiting it, it can be entered
onto the title register for that land by means of a notice. It is the responsibility of the person who
enjoys the interest to apply to the registrar for the entry of a notice on the register. The notice will
go onto the charges register.
Three types of notice;
1. An agreed notice Registered proprietor agrees to the entry of the notice or the registrar is
satisfied with the claim.
2. Unilateral notice Entered onto the register without the consent of the registered proprietor.
However, the owner is informed about the entry and can apply for a cancellation of the
notice. If the person claiming the benefit of the right does not object to the cancellation of
the notice within a specified time it will automatically be cancelled.
3. Automatic notice This relates to those interests requiring registration under S.27 for eg, the
grant of a legal charge. These will be automatically protected by the registrar by an entry of
a notice on the register.
S.33 LRA 2002 lists 5 interests that cannot be protected by the entry of a notice. The ones I need to
know are;
1. The interest of a beneficiary under a trust of land. This is extremely important to
remember
2. A legal lease granted for a term of 3 years or less cannot be entered on the register by a
notice. There is an exception in the case of leases which are required to be registered
irrespective of length under S.27 LRA 2002
3. A restrictive covenant made between a landlord and a tenant cannot be entered on the
register by a notice.
If not one of these exceptions, then it can be entered onto the charges register as a notice.
Common examples are; estate contracts, equitable easements, home rights and restrictive covenants
in freehold land.
What is the effect of entering a notice on the register?
S.29 LRA 2002 provides that once an interest has been entered onto the charges register by means
of a notice it is binding on future owners of the estate. The notice confers priority which means that
it can be enforced against a new registered proprietor.
The restriction is the second way to enter an interest onto the title register. This is governed by
sections 40 to 47.
Restrictions are in the Proprietorship register. They have a very different role and purpose to

notices. A restriction cannot be used to protect an interest which is capable of protection by a notice.
The most common way to come across restrictions is in relation to the interest of a beneficiary
under a trust of land. A restriction is the only way to protect the interest of a beneficiary under a
trust of land.
But what is a restriction? It is basically restricts the way in which the registered proprietor can deal
with their land.
S.41 defines a restriction as an entry in the register regulating the circumstances in which a
disposition of a registered estate may be the subject of an entry in the register. This means the
restriction operates to prevent a dealing with the estate, such as a transfer to a new owner, from
being registered which does not conform to the terms of the restriction.
A common restriction is to require any payment of capital monies to be paid to a trust corporation or
at least two trustees, this ensures that overreaching occurs where land subject to a trust is sold. It is
not the case of once a restriction is on the register the interest of a beneficiary is binding on all
future owners of the estate.
Overreaching interests
These are interests which will bind the purchaser even though they do not appear on the title
register. As the point of the title register is that you should be able to find out the rights and interests
affecting an estate in land by examining the register, therefore these are found to be unsatisfactory
in modern conveyancing.
These are set out in Schedule 1 and schedule 3 of the LRA 2002.
Schedule 3 relates to 'registered dispositions', which means dealing with an estate which is already
registered. Eg, the sale of a registered freehold estate would be a 'registered disposition'
Schedule 3 Paragraph 1 states that a legal lease granted for a term equal to or less than 7 years is an
overriding interest. This is fairly straightforward but there are exceptions
One of these exceptions is that of a reversionary lease, which takes place more than 3 months in
advance. This is common amongst students. This type of lease is required to be registered with its
own title under S.27 LRA 2002.
Schedule 3 Paragraph 2 identifies an extremely important overriding interest, this is an interest
belonging to a person in actual occupation of the land. This helps to think of it as an equation,
- a right in the land + actual occupation = an overriding interest
The first component is that a person must have a proprietary interest in the property, it cannot be a
mere personal interest such as a bare license even with actual occupation. Nor does actual
occupation in the land itself create a proprietary right to occupy. You must first work out what
interest someone has before considering whether it is capable of being overriding interest.
These might include things like; a legal lease, equitable lease, the interest of a beneficiary under a
trust or even an estate contract such as an option to purchase.

Section 31(10) of the Family Law Act 1996, states that home rights cannot be an overriding interest.
Even though this is a proprietary interest must always be entered as a notice on the register in order
to bind a purchaser.
If someone has a proprietary interest in land it may constitute an interest an overriding interest
under schedule 3 para 2 if the person is in actual occupation of the land.
If it is in actual occupation
Some of the cases defining actual occupation are before LRA 2002, but are still relevant.
1. Where the person claiming the interest is in actual occupation and that occupation would be
obvious on a reasonable inspection or
2. Where the person claiming the interest is in actual occupation and the person to whom the
disposition is made has actual knowledge of the interest
Williams & Glyn's Bank v Boland per Lord Wilberforce The words actual occupation are words of
plain English and should be interpreted as such. It is evident by a physical presence in the house,
rather than a mere entitlement at law.
Abbey National Building Society v Cann per Lord Oliver - No more than merely preparatory steps
leading to the assumption of actual residential occupation. Actual occupation requires some degree
of permanence and continuity.
Lloyds Bank v Rosset per Lord Bridge The relevant date for actual occupation to protect an
interest is the date of the transfer, not the date of registration. The wording of Sch 3 Para 2 has cast
doubt on this decision. Obiter statements suggest that occupation by a representative may be
sufficient.
Temporary lapses in occupation;

Linklending v Bustard Per Lord Justice Mummery A person receiving long term residential
medical care may be regarded as still in actual occupation. In this case, she was prevented
from returning home due to her mental health.
Chhokar v Chhokar Per Lord Justice Cumming-Bruce Shorter stays in hospital may also
be regarded. She was having a baby. Belongings in the house and intention to return is
sufficient.
Malory Enterprises Ltd v Cheshire Homes Ltd Per Lord Justice Arden Whether there was
actual occupation at the time is essentially a question of fact for the judge. The judge had to
consider by other means, as there were derelict buildings, denoting occupation such as
boarding up the windows and fencing the site, to keep trespassers and vandals out.

Sch 3 Para 2 also lists the circumstances where a persons proprietary rights in actual occupation of
the land will not amount to an overriding interest.

There will be no overriding interest if the person with the interest was asked whether they
had an interest in the land before the disposition and they failed to disclose their interest
when they could reasonably be expected to do so. This is common sense, as if the purchaser
asks the occupier if they have an interest in the land and they say no, then they cannot turn
around and state that they do. This would be unfair and is covered in Para 2.

There will be no overriding interest if the persons actual occupation was not obvious upon a
reasonable inspection of the land and the purchaser did not know about the existence of the
proprietary interest. It is the persons occupation that should be obvious not their interest in
land.

Schedule 3 Paragraph 3 relates to implied legal easements.


As an expressly created legal easement must be registered under S.27 LRA 2002 to operate at law,
before it is registered it will be equitable only and cannot be an overriding interest. An easement
must be legal and it must be impliedly created. Only those that comply with the provisions in Sch 3
Para 3 will be protected as overriding interests. Though there are 3, only one needs to be satisfied.
1. The legal easement will be an overriding interest if the purchaser has actual knowledge of
the easement.
2. The easement will bind if the easement is patent, which means it would be obvious upon a
reasonable careful inspection of the land.
3. The final condition is that the easement has been exercised within one year before the
disposition.
The issue of priority in registered land.
Clearly a purchaser needs to know about any 3rd party rights before they go ahead and purchase the
land.
S.29 LRA 2002 is a very important provision.
S.28 LRA 2002 states that All interests that were in existence before the registered estate was
transferred to the new owner will be binding on that new owner.
S.29 LRA 2002 That a person acquiring a registered estate for valuable consideration and who
completes their purchase by registration takes the land subject ONLY to the interests that are
protected by a notice on the register and interests that are overriding.
It is important to note that S.29 only applies to a transferee who BUYS a registered estate WHO
GIVES VALUABLE CONSIDERATION for that purchase, and WHO COMPLETES THEIR
PURCHASE BY REGISTERING THEMSELVES AS THE NEW OWNER.
It does not apply to those who have been given land by way of gift, so S.28 would apply.
Overreaching
Overreaching is different to overriding interests.
Overreaching applies to the interest of a beneficiary under a trust. It is a means by which a
purchaser of land can ensure that after they have bought the land they will not be bound by any
beneficial interests under a trust. Section 27 LPA 1925, states that for overreaching to occur the
purchase monies to at least two trustees or a trust corporation. As long as these criteria are met
overreaching converts the interests of equitable owners from an interest in land to an interest in the
capital monies such as the sale proceeds. This means that the purchaser does not need to worry
about the beneficiaries under a trust claiming a share in the equitable ownership of the land.

The beneficiary cannot claim any interest in the land, but does have a claim in the sale proceeds
from the trustees. If the beneficiary is dissatisfied with the money raised from the sale, or feel that
the trustees have embezzled money, their only remedy is in damages against the trustees and cannot
claim the land from the purchaser.
This is especially reassuring due to the rule 'Overreaching beats overriding'. Which means that if
overreaching occurs, then this trumps any potential overriding interest the beneficiary may normally
be able to claim. This is illustrated in City of London Building Society v Flegg.
Co- Ownership
Co-ownership describes a situation where 2 or more people have an interest in the ownership of the
same property at the same time.
These days many people buy a house together with their partner and so co-ownership is extremely
significant in practice. There is a statutory requirement that whenever property is co-owned there is
a trust.
Ss34 - 36 LPA 1925 as amended by Trusts of Land and Appointment of Trustees Act 1996
(TOLATA) imposes a trust on all co-owned property.
If someone buys a house together, they will both be the trustees and the beneficiaries. As trustees
they will own the legal title and have various managerial type responsibilities within the trust. As
beneficiaries they have the right to enjoy the property and have ownership of its monetary value.
Statute limits who can be a trustee, ie the legal owners.
S.34(2) of the LPA 1925:
There can be no more than 4 trustees. This is to simplify the conveyancing process as the trustees
are the ones who need to sign any transfer document in the future when the house is sold, so the
fewer the better.
In addition S.34 prohibits anyone under the age of 18 from being a trustee. If more than 4 people
buy a house, the first 4 names in the transfer deed, that are over 18, will be the trustees, holding the
land on trust for themselves and the remainder on trust.
TOLATA
Stands for Trusts of Land and Appointment of Trustees Act 1996.
TOLATA came into force on 1st January 1997 and applies to ALL trusts of land.
S.1(1) (a) states : that a trust of land means any trust property which consists of, or includes land.
This includes, express and implied trusts, and where the trustees and beneficiaries are one and the
same people as in a couple owning their own home.
TOLATA sets out what powers the Trustees have.

S.6(1) gives the Trustees 'all the powers of an absolute owner'. This lets them do anything that an
outright owner could do including selling the property, lease the property and mortgage it.
However, these powers are only available to them so as to enable them to exercise their function as
trustees. S6 (5) of TOLATA provides that he trustees must have regard to the rights of the
beneficiaries.
And S.6 (6) provides that they must not act contrary to any rule of law or equity.
Trustee's cannot profit from the trust.
Specific restrictions

S.10 states that an expressly created trust can impose a requirement for consent from named
parties to any dealings with the land. This could be consent from anyone, but usually it is
consent from a beneficiary.
S.11 creates a duty on trustees to CONSULT with the beneficiaries so far as is practicable
and give effect to the wishes of the majority by value.

The majority by value is refers to what constitutes the largest beneficial interest. However, should
the Trustees disagree with what the beneficiaries want, they do have the power to override this if
they think the beneficiaries wishes are not consistent with the general interest of the trust.
Rights of the beneficiaries
S.12 TOLATA
General right to occupy the trust land for any beneficiary who has an interest in possession.
This section then identifies when that right arises.
Thus the purposes of the trust must include making the land available for occupation by the
beneficiary. It would be, if the house was bought specifically for the beneficiary to live in.
Alternatively, even if the land wasn't bought for this reason, the beneficiary may be able to occupy,
if the land is in fact available for occupation.
Also, if it is available for occupation, it must be suitable for occupation. This would not be the case,
if the property was derelict.
S.13 TOLATA Sets out some conditions on the right to occupy.
S.13(1) The trustees can still exclude or restrict one or more of the beneficiaries from occupying.
However, they cannot ban them all. They can only exercise this power if they are doing so
reasonably.
Then under S.13(3) the trustees can impose reasonable conditions upon them, such as making them
pay outgoings on the property. The combined effect of S.13(3) and (6) is that the occupying
beneficiaries may have to pay some compensation to the excluded beneficiary.

TOLATA and dispute resolution


Under S.14(1) an application for a court order to resolve a dispute can be made by any trustee of
land or any person who 'has an interest in property subject to a trust of land'. This could be a
beneficiary or even someone who has a charge on the property like a mortgagee.
Once someone makes an application to the court, S.14(2) provides that the court can make any
order it thinks fit in relation to the exercise by the trustees of any of their functions; or for the
purpose of declaring the nature or extent of any person's interest.
An illustration of the former would be forbidding a trustee to sell the house.
S.15(1) TOLATA gives guidance to the court on matters it must consider in deciding an application
under S.14.

Provision a) refers to the intentions of the person(s) who created the trust
Provision b) refers to the purposes for which the trust property is held. Stott v Radcliffe
illustrates this quite well even though it was decided before TOLATA. An unmarried man
and woman bought a home for them both together or for the sole survivor if one was to die
before the other. The man died and his wife wanted the house sold, however as the woman
whom he had bought the house with did not, the court refused to grant a order for sale as the
intention was to provide a home for the survivor.
c) Refers to the welfare of any minor who is, or might reasonably be expected to be in
occupation.
d) Refers to the interests of any secured creditor of any beneficiary.
Secured creditors are people who have lent you money and you give them an interest in the
property that you own.

Where there is a dispute over occupation, then under S.15(2) the court must also consider the
wishes of unaffected beneficiaries and in relation to any other disputes under S.15 (3) the court
must consider the wishes of the beneficiaries who are of full age and entitled to an interest in
possession.
1. TOLATA does not direct the court to give more weight to one factor than to another.
2. S.15 uses the word 'include' therefore the court is free to consider other additional factors
as well such as the health of a beneficiary.
Though the act gives no guidance to the weighting of each competing factor, the courts are likely to
give precedence to the commercial interests of the creditor, rather than the family's interests. As in
the Fred Perry Case.
In bankruptcy, the trustee will make an application under S.14 TOLATA for a sale of the property
because the beneficiaries creditors will want the property sold in order to get back some of what
they were owed. Once this has been done, the courts will not look at S.15 criteria, as S.15(4) makes
them inapplicable.
The courts would then look at S.335(A) of the Insolvency Act 1986.
(1)Any application by a trustee of a bankrupts estate under section 14 of the Trusts of Land and
Appointment of Trustees Act 1996 (powers of court in relation to trusts of land) for an order under
that section for the sale of land shall be made to the court having jurisdiction in relation to the

bankruptcy.
(2)On such an application the court shall make such order as it thinks just and reasonable having
regard to
(a)the interests of the bankrupts creditors;
(b)where the application is made in respect of land which includes a dwelling house which is or has
been the home of the bankrupt or the [F3bankrupts spouse or civil partner or former spouse or
former civil partner]
(i)the conduct of the [F4spouse, civil partner, former spouse or former civil partner], so far as
contributing to the bankruptcy,
(ii)the needs and financial resources of the [F4spouse, civil partner, former spouse or former civil
partner], and
(iii)the needs of any children; and
(c)all the circumstances of the case other than the needs of the bankrupt.
(3)Where such an application is made after the end of the period of one year beginning with the first
vesting under Chapter IV of this Part of the bankrupts estate in a trustee, the court shall assume,
unless the circumstances of the case are exceptional, that the interests of the bankrupts creditors
outweigh all other considerations.
(4)The powers conferred on the court by this section are exercisable on an application whether it is
made before or after the commencement of this section
The harshness of this was shown in Re Citro 1991 Where two brothers were made bankrupt at the
same time, and both owned a home and each had a wife and three kids. Nevertheless an order of
sale was made to be postponed no longer than 6 months.
In Fred Perry (Holdings) Ltd v (1) Ivan Genis (2) Ayelet Haim Genis (unreported) 1 August 2014
(High Court), the court held that unsecured debt converted to secured recovery takes priority
over family interests to uphold effective debt recovery.
Background
The claimant obtained judgment against Mr Genis for selling counterfeit goods under the
claimants brand. Judgment was secured by way of a charging order on the family property solely
owned by Mr Genis. Mrs Genis claimed a beneficial interest in the property and had further
protected her interest by way of a Home Right under s.30 Family Law Act 1996. Mr and Mrs Genis
resided at the property with their two young children. The claimant sought an order for sale of
the property.
Held
Although the legal estate was vested in Mr Genis alone, the property was held beneficially by
both of the Defendants. Therefore the court referred to the provisions of s.15(1) Trusts of Land
and Appointment of Trustees Act 1996, specifically being the purpose of the property, the welfare
of any minor and the interest of the secured creditor. No guidance was provided as to the relative
weight of each consideration.
The following useful points arise from the judgment in this case:

The relevant case authorities suggested a tension between the legitimate pursuit of a
debt and the rights of innocent victims effected by the recovery.

On balance, the commercial interests favoured the residential security of the family (Bank
of Ireland Home Mortgages Ltd v Bell [2001] 2 All E.R. (Comm) 920 considered).
The absolute right of a trustee in bankruptcy to obtain an order for sale of a bankrupts
property under Insolvency Act s.335A suggested an overriding consideration of financial
interests over family interests in a statutory context.
An order for sale could not be said to disproportionately interfere with the rights of the
family under ECHR since the property was the only asset which could satisfy the debt.

Mrs Genis could not rely on absolute protection from interference with the family home,
afforded by the Home Rights notice since this would be unfair to a spouse who had not
registered their right over a property. The test for a charging order would have to be
s.15(1) Trusts of Land and Appointment of Trustees Act 1996.
The circumstances of each case will be the determinant aspect of the judgment.
Although the claimant succeeded, the court tempered the effects on the family by
deferring the order for one year.
Comment
This decision confirms that the court views an order for sale of a matrimonial asset as a draconian
weapon in a secured creditors armoury. Secured creditors should only deploy it as a step of last
resort and if the equity position justifies it. The rights of beneficial owners, minors or any other
innocent third party connected with the property may not prevent such an order, if the secured
creditor can demonstrate no other alternative and a good chance of recovery.

Types of Co-ownership
There are two types of co-ownership, they can be joint tenants or tenants in common.
Although the word tenant is used, it has nothing to do with leases.
Joint Tenancy
Each owner is seen as entitled to the whole interest in the property. There is no concept of shares in
the property, instead the joint owners are seen together as one unit which owns the totality of the
shares.
There is one important consequence of co-owning as a joint tenant and that is it's impact upon
death. This is called the 'right of survivorship'.
eg. If someone dies as a joint tenant, their interest is extinguished upon death. He has no distinct
share in the property which he could pass by will or intestacy. The other party would simply
become the sole owner.
Tenancy in Common
In this way, each co-owner is entitled to a specific share, one that is separate from the other owners.
Tenants in common can decide whether to have equal or unequal shares. This becomes relevant if
the property is sold. The shares may or may not reflect each co-owners contribution to the purchase
price. It could still be agreed that someone who contributed less would receive more.

The right of survivorship does not apply to tenants in common, as each person has their own distinct
share in the property.
The trustees who own the legal estate can only be joint tenants, this is the impact of S.1(6) of the
LPA 1925;
'A legal estate in land is not capable of subsisting or of being created in an undivided share in land'
'Undivided share' is the key word here. A TIC can have a share in the land but a JT cannot. JT own
the whole of the property concurrently.
However, the beneficiaries can be either JT or TIC.
If two people choose to be TIC, then they will still be JT with the legal estate, so when one dies, the
other will become the only trustee due to the right of survivorship, but the equitable interest can
pass to whoever they choose. The original purchaser will therefore be a trustee for themselves and
whoever the interest was passed to.
Four Unities
This is the name given to the four requirements which must be satisfied for a JT to exist.
The first is the unity of possession This means that each co-owner is equally entitled to possession
of the whole property.
The second is the Unity of Interest This means that each co-owner must have an identical interest
in the property. Eg, they must both own a freehold estate.
The 3rd is the Unity of Title Co-owners must both acquire the interest from the same document.
Could be no unity of title if they had each signed separate deeds.
The 4th is the Unity of Time Co-owners must all receive their interest in the land at the same time.
All 4 unities must exist for there to be a JT. If one is missing there will only be a TIC. They usually
do exist, due to the standard conveyancing forms, the TR1 form. Thus by using this one document,
co owners will acquire the same title, at the same time, using the same document.
For a TIC, there needs to only be one unity, and that is the Unity of Possession. It's fine if the others
exist too, but Possession is the only one that is needed. This is only needed because if one of the coowners could exclude another from part of the house, that would not be co-ownership. Each person
would be a sole owner of part of the property. This is why we say that TIC having 'undivided'
shares.

How to work out if its a beneficial JT or TIC


The first thing that must be checked is:

Whether all 4 unities were present when the co-owners bought the property as the beneficiaries can
only be JT if the 4 unities are present. However, they are usually present in the TR1 form.
Next;
Whether the co-owners said they wanted to be JT or TIC. Usually, they make a written statement
stating which they would be. If they do this, the decision in Goodman v Gallant says it would be
binding on them unless it was made fraudulently or by mistake. The TR1 form has a check box for
this. They can decide to be;

Beneficial JT
Benefical TIC in equal shares
Beneficial TIC in some other proportion

Generally there is a record.


If there is no record, equity presumes there should be a TIC, the most common situations are;
Parties have made unequal contributions to the purchase price
co-owners are in a business partnership together
However, these are just presumptions. And can be rebutted with evidence that they intended to be
JT.
The presumption of TIC because of unequal contributions does not apply in the family home.
Following Stack v Dowden.
Severance of beneficial JT
This is the method of changing from a JT to a TIC where there is no right of survivorship.
We say he can sever his beneficial JT to become a TIC. Both statute and common law permit
severance.
However, there is a golden rule No one can sever their legal JT. In the same way it is not possible
to create a legal TIC.
This is laid down in statute by S.36 (2) LPA 1925 to preserve simplicity of conveyancing.
THE TRUSTEES CAN NEVER SEVER THE LEGAL JOINT TENANCY
In Goodman v Gallant it was held that; The value of the beneficiary's share once severance has
taken place, will not simply reflect the original contributions to the purchase price. It stated that the
co-owners became entitled to equal shares of the property after severance.
Where there are 3 or more co-owners, if one chooses severance, then it is only he that becomes a
TIC, the other 3 will remain JT. Eg, if four people own a house as JT and one chooses severance, he
will become a TIC with a share, while the others are still JT with a interest.

Statutory severance
S.36(2) LPA 1925
Requires the joint tenant who wants to sever to give a written notice to the other JT's.
If there are more than 2 JT's he must give the notice to each of them but he does not need to consult
them first or obtain their consent.
Problems arise where not all notices have been professionally drafted. Case law has clarified what is
required for this style of severance.
The first issue is the form of the notice.
In the case of Re Draper's Conveyance, it was held there is no prescribed form that is needed for the
notice.
The second issue is the content itself. In Harris v Goddard it was held there must be an immediate
intention to sever the beneficial JT.
This has two elements to it;
1. Must show an intention to sever immediately.
2. Must make clear they wish to sever and become a TIC, if this is not expressly stated they
should make reference to a share in the property.
The 3rd issue is that of acceptable ways to deliver the notice. The act of 'giving the notice' or
'service' of the notice.
Concerned with WHO must be given the notice and WHAT method of delivery is acceptable.
S.36 (2) LPA 1925 states that service of the notice MUST be on ALL of the other JT's.
S.196 (4) LPA 1925 states that it can be sent by POST and 'notice can be served by registered or
recorded delivery to the last known place of abode or business of the addressee'
whilst S.196 (3) says the notice can be left at, or affixed to, the last known place of abode or
business of person being served. This is talking about delivery in person. Though in Kinch v
Bullard, it was held it does also include service by post. Thus ordinary 1st class post was acceptable
if it could be shown the postman had in fact delivered it. Even though the recipient never read it.
Lord Neuberger also commented on whether a change of mind could prevent severance
happening, and held obiter that 'if the sender had communicated their change of mind after the
notice was sent but before it was received then it possibly could.'
This case and, Re: 88 Berkeley Road, both show that provided the notice is correctly served, it
doesn't actually have to be received by the addressee.

Common Law severance


The case of Williams v Hensman, identified three methods of severing the beneficial JT

By an act operating on one's own share, or


Through mutual agreement, or
Through mutual conduct

1. Operating on one's own share


Occurs when a co-owner deals with their potential share in some way, an example of this would be
if a co-owner sold his interest. This would result in the seller and the purchaser becoming TIC.
The action must be capable of being legally enforced;
Like selling an interest
Making a legally enforceable contract to sell the interest; or
the co-owner granting a mortgage over the interest; or
the co-owner becoming bankrupt, this automatically severs a beneficial JT irrespective of
the wishes of the co-owner.
Writing a will is not a legally enforceable action so this itself will not constitute severance.

2. Through mutual agreement


This means that all of the JT's agree that one or more of them will hold their interest as a TIC.
The agreement does not have to be legally enforceable established in Burgess v Rawnsley.
3. Mutual conduct or 'Mutual course of dealing'
ALL of the JT act in a way which makes clear they treat themselves as TIC. There is no actual
agreement between them, but they're behaviour shows they are treating the JT as severed.
This method is very rare.
Unsuccessfully argued in Greenfield v Greenfield, the court held it was done for practical reasons.
The conduct must fall short of an express or implied agreement, but must show an unambiguous, common
intention to sever: Greenfield v Greenfield (1979). For example, where the co-owners execute mutual wills it is
not the wills themselves which sever (severance cannot be effected by will); but rather the intention which
underlies them shows the decision to sever.

In Burgess v Rawnsley, Lord Denning thought that unsuccessful negotiations that did not result in
an agreement could amount to severance by mutual conduct. However, the rest of the COA did not
agree.

Leasehold Covenants
A covenant is a promise made by one party, the covenantor, for the benefit of another party, the
covenantee.
Leasehold covenants are the terms agreed in a lease which relate to the parties obligations to each
other in their capacity as landlord and tenant. Some are written expressly into the lease document by
the parties and some are implied into the lease by common law or statute.
Common express statutes
Firstly, there is a covenant to pay the rent, usually there is also a rent review clause which allows
the rent to be reviewed and amended.
Another common one is a user clause, which means that the tenant agrees to use the property in a
particular way eg, not for business.
Qualified covenant against assignment is where the tenant covenants not to assign or sublet the
lease without the prior consent of the landlord. This is different to an Absolute covenant against
assignment which does not allow subletting or assignment.
Where there is a qualified covenant against assignment clause in the lease. S.19 of the Landlord and
Tenant Act 1927 implies a proviso into the lease that the landlord cannot unreasonably refuse
consent to assign. Unfortunately there is no guidance on what would constitute reasonable grounds
for refusal.
However, if the landlord does unreasonably refuse consent, the tenant can assign the lease and this
will be an effective assignment. This is risky as if it is deemed reasonable grounds for refusal, then
this assignment will be deemed a breach of covenant for which the landlord can seek a remedy.
Section 1 of Landlord and Tenant Act 1988 imposes a duty on landlords to reach a decision
within a reasonable time and to give reasons for refusing consent. This clearly helps the tenant work
out whether the landlord is being reasonable in refusing consent.
Common Landlord Covenants
It is common for the landlord to expressly covenant to allow the tenant quiet enjoyment of the
premises. In any event this is implied into every lease which does not account for it expressly. This
does not necessarily refer to noise but more an uninterrupted enjoyment of the property.
In Owen v Gadd, the landlord erected scaffolding outside a shop which was let by his tenant, this
amounted to a breach.
A covenant is implied into all leases that a landlord will not derogate from his grant. This means he
will not take away what he has given. This is relevant where a tenant rents land for a particular
purpose, but then the landlord acts in such a way that the original use cannot be performed.
This was shown in Aldin v Latimer Clark, Muirhead & Co. Where the tenant was a timber merchant
and the landlord interfered with timber wood drying sheds and prevented the tenant from doing the
only thing he needed to use the land for. The court held he was in breach.

Repair covenants could go either way, for example in a short 3 year lease, it is likely that the
landlord may be responsible for any structural repairs such as a leaking roof, while the tenant
covenants to carry out relatively minor internal repairs like doors etc. However, for a long lease
such as a 99 year lease, it would be likely that the larger repairs would be undertaken by the tenant.
The same approach is usually taken with regards to home insurance.
Enforcement of leasehold covenants
Every covenant has a benefit and a burden. Eg with a tenant repair covenant, the landlord has the
benefit of that covenant and can sue. The tenant has the burden of that covenant and is likely to be
sued if he was to breach it.
What would happen if the owner were to sell the freehold reversion, or the tenant were to assign his
lease to someone else?
It's necessary to work out if the benefit and burden has been passed on. The repair covenant for
example could only be repaired if the benefit passed to the new landlord and the burden was passed
to the new tenant.
There is one set of rules for leases before 1st January 1996 and a different set of rules for after.
IT IS THE DATE WHERE THE LEASE WAS ORIGINALLY GRANTED THAT MATTERS, NOT
WHEN IT WAS LATER ASSIGNED.
Even if a lease was assigned in 2005 but was granted in 1995, it would still be an old lease.
Landlord and Tenant Act (Covenants) 1995 governs the leases granted after 1st january 1996, but
does not have retrospective effects.
Enforcement of old leases Original Parties
Between the original landlord and the original tenant, it is fairly straightforward, as there is privity
of contract between them and while the tenant holds the leasehold and the landlord holds the
freehold reversion they can enforce all the covenants against each other.
Section 79 LPA 1925 states that, a covenant relating to any land of a covenantor shall be deemed to
be made by the covenantor on behalf of himself and his successors in title and those persons
deriving title under him. This means the original landlord and the original tenant remain liable for
breaches of covenants by their successors in title for the whole duration of the lease.
RPH Ltd v Mirror Group Newspapers, is an example of the harshness of this rule. They had to pay
over 1 million even though they had assigned the lease 20 years earlier.
The rule of continuing liability in the old leases only applies to the original parties and not their
successors in title. If someone assigned a lease to another person, and then the 2nd person assigned
the lease further, the original tenant would still be liable, not the 2nd one.
When assigning the lease or selling the freehold reversion, the seller should include an indemnity
clause from their immediate assignee. There is an implied indemnity in registered land under S.134
and Schedule 12 of LRA 2002, or the assignment documentation can expressly include one. An
indemnity covenant allows the party to recover damages in the event of having to pay out due to
S.79.

Enforcement of new leases


These are the rules regarding new leases which are granted on or after 1st January 1996 and are
governed by the Landlord and Tenant Act (Covenants) Act 1995.
Examining the position of those who acquire an assignment of the lease or who buy the freehold
reversion. The position is the same for both the new tenant and the new landlord.
Section 3 Landlord and Tenant (Covenants) Act 1995
This replaces the rule established in spencer's case and sections 141 and 142 of the LPA 1925 which
required the test of the 'touch and concern' of the land.
The new rules established by Section 3 is;
'the benefit and burden of ALL the covenants in the lease pass on assignment of the lease or the sale
of a freehold reversion'
There is one exception wherein, if there are any covenants expressed to be personal, these will not
pass to the successors.
So if there is a repair covenant in place with the original landlord and tenant, and the lease is
assigned to another, and/ or the landlord sells the freehold reversion, this repair covenant will pass
onto the new tenant and the new landlord would be able to enforce it. This is because a repair
covenant is not considered personal.
The original parties and new leases
The enforcement of leasehold covenants between original parties in new leases, is still governed by
privity of contract and as a result is the same as old leases.
However what happens if the original parties sell the freehold reversion or assign the lease?
One of the LTCA 1995's aims was to reform the law relating to continuing liability of the original
parties established in the old leases by S.79 LPA 1925 and such cases like RPH ltd v Mirror Group
Newspapers
It applies differently to landlords and tenants.
Tenants
Section 5 LTCA 1995 sets out the general rule. This section provides that the original tenant is
automatically released from continuing liability. Meaning once the lease has been assigned he is no
longer liable for breaches by his assignee.
However, there are two important exceptions to the general rule under S.5 and BOTH of which are
triggered where the lease contains a qualified covenant against assignment. These can be found in
S.11 and S.16 of the Act.

S.11 applies to excluded assignments. An excluded assignment is one made in breach of covenant
against assignment. Eg, if there is an express qualified covenant against assignment and the lease is
assigned without the consent of the landlord, then this assignment is made in breach of covenant.
This is an excluded assignment under S.11
This is an exemption to S.5 so the original tenant would remain liable for breaches of covenant by
his assignee. However, the original tenant will be released from liability on the next assignment
which is not an excluded assignment, so it is possible to be released from continuing liability.
The next exemption is that of Authorised Guarantee Agreements under S.16.
It is common for consent for assignment to be given to the original tenant by the landlord subject to
an extra written agreement where the tenant agrees to guarantee the performance of the leasehold
covenants by his immediate assignee. This is an Authorised Guarantee Agreement. The tenant is
only liable for breaches by his immediate assignee.
The landlord could ask for an AGA from any future tenant who wished to assign the lease.
Landlord
Under S.6 LTCA 1995, the landlord may be released from his covenants on the sale of the freehold
reversion but this release is not automatic like the tenants release under S.5.
Instead, he must follow the procedure in S.8 LTCA 1995, by which he must serve a notice on the
tenant before or within 4 weeks of the sale of the freehold reversion, notifying the tenant of sale and
requesting a release from the covenants set out in the lease. The landlord will then be released if the
tenant consents, fails to object or a court orders a release on behalf of the landlord.
These next sections apply where there is some rent arrears to be paid by the current tenant and
whether the former tenant will have to pay them. This will apply to old leases due to continuing
liability under S.79 LPA 1925, however will only arise in a new lease where the original tenant
remains liable due to an excluded assignment or an Authorised Guarantee Assignment.
Section 17 of LTCA 1995, states that a former tenant will not be liable to pay any fixed charge such
as rent or a service charge owed by the current tenant unless the landlord serves a notice on the
former tenant within 6 months of the fixed charge becoming due. This alerts the former tenant to the
charge and allows them to take some action to stop the arrears accruing.
Section 19 of LTCA 1995, provides that if the former tenant pays the charge in full, he is then
entitled to what is called an 'overriding lease'. This is a lease carved out of the landlords reversion
which exists concurrently with the existing lease and makes the former tenant the immediate
landlord of the current tenant and can make use of remedies available to landlord's such as the
remedy of forfeiture.

Successors in title Old leases


Tenant
Can those taking an assignment of the lease sue and be sued for breaches of leasehold covenants?
Spencer's case 1583 provides a three pronged test for working out whether a benefit or burden has
passed to the assignee.
1. The lease in question must be a legal lease assignment must also be made by deed,
2. There must be a privity of estate between the party suing and the party being sued.
3. The covenant in question must touch and concern the land.
2. Privity of estate
This simply means that there must be a current relationship of landlord and tenant between the party
who is suing and the party being sued in respect of that same leasehold estate.
Between the original landlord and tenant there is both privity of contract and estate, however in
respect of a successor in title to the landlord and successor in title to the tenant there is no privity of
contract, but there is privity of estate due to their current relationship as landlord and tenant.
3. Touch and concern the land
The best working definition of this is found in P & A Swift Investments v Combined English
Stores, Per Lord Oliver;
Defined a 3 point definition of 'touch and concern' ;
1. A covenant given by a tenant will only touch and concern the land if it benefits the landlord
only while he holds the freehold reversion. Therefore if he sells it, the covenant would no
longer be a benefit to him.
2. It must affect the nature, quality, mode of use or quality of the land. This is again
emphasising that the covenant affects the land and is not merely personal. For example if the
landlord wants to enforce a repair covenant against the tenant, this clearly relates to the
quality and value of the land.
3. The final part reiterates that the covenant cannot be personal in nature.
Can successors in title to the landlord sue and be sued?
The answer is found in S.141 and S.142 LPA 1925. Which provide that they can sue or be sued
provided that the covenants 'have reference to the subject matter of the lease'. S.141 relates to the
passing of the benefit of a covenant and S.142 relates to the passing of the burden of a covenant but
in both cases the same test applies.
The crucial words are ' have reference to the subject matter of the lease' and the courts have held
that this is effectively the same test as the one laid down by Lord Oliver in the swift investments
case relating to the 'touch and concern of the land'.

Landlords remedy of forfeiture


Forfeiture is a draconian remedy as it enables the landlord to repossess the property and terminate
the lease prior to the contractual end date. This clearly has serious consequences, so there a
numerous safeguards in place to prevent abuse.
Firstly, it is not an automatic remedy that applies to every lease. It must be expressly included in the
lease before the landlord can rely on it. So a solicitor must always read the lease carefully before
advising a landlord to make use of this remedy. It is also known as a re-entry clause, and the clause
itself is unlikely to use the word forfeiture. Look instead for a reference to the landlord being able to
re-enter the premises and determine the lease if a tenant breaches covenant.
Another important preliminary point is that the landlord must ensure that the tenants breach of
covenant has not been expressly or impliedly waived. There are two requirements to an implied
waiver.
1. The landlord must have knowledge of the breach of covenant, and
2. The landlord (or his agent) must then take some action which confirms the continued
existence of the lease. The most common instance is demanding or receiving rent. As was
found in the case of Segal Securities v Thoseby
It is important that the landlord must have knowledge of the breach when the rent is demanded or
received.
It can be problematic for landlords that they are bound by the actions of their agent. So if the
landlord knew of a breach and the landlords agent demanded rent then this will amount to a waiver
by the landlord.
If there are multiple breaches, then it is only for those that he knew about when he demanded the
rent, if he finds out others after and hasn't charged or received rent then he could seek the remedy.
In Greenwich v Dicreet Selling Estates, the COA noted that 'in the case of a continuing breach the
right to forfeit arises afresh the very next day and can then be exercised.'
This is because a waiver only applies to past breaches, but not to future ones. A breach of a tenant
repair covenant is a continuing breach, and as the disrepair would still be there the next day even
after the landlord had demanded rent, he could pursue the remedy.
There are two different procedures as to the non payment of rent than there is for every other type
of breach.
Tenant in arrears with the rent
The landlord must first make a formal demand for the rent. However, where the lease expressly
dispenses with the need for a formal demand, which is common, this is not needed.
Additionally, if the rent is more than 6 months in arrears then Section 210 of the Common Law
Procedure Act 1852, dispenses with the need for a formal demand.

All other breaches


For any other breach of covenant, the landlord must serve an S.146 notice on the tenant. The aim of
this is to give the tenant a chance to remedy their breach rather than face eviction.
Section 146 LPA 1925 lists 3 specific requirements.
1. The notice must specify the breach of covenant complained of. This is IMPORTANT.
In Akici v Butlin Ltd, the landlord served an S.146 notice on a tenant stating the tenant had
'parted with possession' of the premises in breach of a covenant not to 'part with possession
or share possession'. The court held the tenant was sharing possession and therefore the
notice was not effective.
2. The landlord must ask for the breach of covenant to be remedied if it is capable of remedy.
This has given rise to problems.
3. The notice must require the tenant to pay financial compensation if the landlord wants it.
This can be excluded from the notice if the landlord doesn't require any.
Returning to point 2 and that the landlord must ask for the breach of covenant to be remedied if it is
capable of remedy. How do landlords decide whether it is capable of remedy? S.146 gives no
guidance.
Case Law
Rugby School (Governors) v Tannahill [1935] 1 KB 87 (Court of Appeal)
Per Greer LJ
Held that breaching this covenant not to use the premises for any illegal or immoral purpose was
not capable of remedy. Dismissed comments by Mckinnon LJ at first instance that a negative
covenant, ie not to do something, could never be capable of remedy.
In, Van Haarlam v Kasner (1992) 64 P. & C.R. 214, Harman J, again the courts were reluctant to
say that all negative covenants are not capable of remedy due to the stigma attached to the property.
'The landlord was entitled to be protected against the slur of being said to be a landlord to a person
who used the flat for criminal activities'. This meant the S.146 notice was valid even though the
landlord had not asked for the breach to be remedied.
The landlord argued that he did not know of the illegal activities at the time he demanded and
received rent off the tenant. However, the court held that he must have known due to the case being
reported widely in newspapers and had therefore impliedly waived his right to forfeit for that
particular breach of covenant.
The court did not need to decide on relief, but did so obiter. The courts discretion under S.146(2)
1925. Even though it was an irremediable breach the court felt it would have been appropriate to
grant relief due to the fact there was still 80 years remaining on the lease. To allow forfeiture would
have been disproportionate damage compared to what was done.
However, in Ropemaker Properties Ltd v Noonhaven Ltd (No.1) [1989] 34 E.G. 40, the court
held that in exceptional circumstances the court may grant relief from forfeiture for a breach
involving an immoral user.

In this case it was a commercial lease, where night clubs were being used for prostitution.
These are some of the reasons why it was held it could grant relief in rare occasions.
Held, that (1) the lease was forfeit; but (2) this was one of those very rare and exceptional situations
where the Court would grant relief from forfeiture for a breach involving immoral user, having
regard (i) to the substantial value of the lease; (ii) to the substantial loss (out of proportion to their
offence or to any damage caused by it which forfeiture would cause to T Co.; (iii) to the fact that the
immoral user had been brought to an end and was unlikely to be renewed; (iv) to the fact that any
"stigma" attaching to the premises was likely to be short-lived; (v) to the fact that any such "stigma"
would not be removed by getting rid of T Co.; (vi) that to grant relief would not be to saddle L with
unacceptable tenants; and (vii) to the poor health of T Co.'s sole director, and to his intention to
dispose of the lease.

Expert Clothing v Hillgate House [1986] 1 Ch 340 (Court of Appeal)


Slade LJ Held there was a distinction between time limits regarding remedies within a
reasonable time limit for 'negative breaches of covenant' such as those in the Rugby
School Case, and breaches of positive covenants such as this one.
'In my judgment, on the remediability issue, the ultimate question for the court was
this: if the section 146 notice had required the lessee to remedy the breach and the
lessors had then allowed a reasonable time to elapse to enable the lessee fully to
comply with the relevant covenant, would such compliance, coupled with the payment
of any appropriate monetary compensation, have effectively remedied the harm
which the lessors had suffered or were likely to suffer from the breach? If, but only if,
the answer to this question was "No," would the failure of the section 146 notice to
require remedy of the breach have been justifiable. In Rugby School (Governors) v.
Tannahill [1935] 1 K.B. 87; Egerton v. Esplanade Hotels, London Ltd. [1947] 2 All E.R.
88 and Hoffmann v. Fineberg [1949] Ch. 245 the answer to this question plainly would
have been "No." In the present case, however, for the reasons already stated, I think
the answer to it must have been "Yes."'

My conclusion, therefore, is that the breach of the covenant to reconstruct was "capable of remedy."
In reaching this conclusion, I find it reassuring that no reported case has been brought to our
attention in which the breach of a positive covenant has been held incapable of remedy, though I do
not suggest that cases of this nature, albeit perhaps rarely, could not arise.
In the result, I would allow this appeal on the basis that, while the defendants have
established no waiver, the plaintiffs' section 146 notice was invalid because the
relevant breaches were capable of remedy.

Savva v Hussein (1997) 73 P & CR 150 (Court of Appeal)


This dealt with a covenant not to display certain types of signs outside of the shop.
Those signs that were allowed, the colour scheme had to be approved by the landlord.
'
Staughton L.J.
In this case the question is whether the breaches, if there were breaches, were capable of remedy. They
amount to doing things without the consent of the landlord. That is what the covenant did not allow.
It is established law in this court that the breach of a covenant not to assign without consent cannot be
remedied. That was decided in Scala House & District Property Co. Ltd v. Forbes. Even then relief from
forfeiture was granted, so that may not be of any great consequence.
In my judgment, except in a case of breach of a covenant not to assign without consent, the question is:
whether the remedy referred to is the process of restoring the situation to what it would have been if the
covenant had never been broken, or whether it is sufficient that the mischief resulting from a breach of
the covenant can be removed. When something has been done without consent, it is not possible to restore
the matter wholly to the situation which it was in before the breach. The moving finger writes and cannot
be recalled. That is not to my mind what is meant by a remedy, it is a remedy if the mischief caused by the
breach can be removed.
In the case of a covenant not to make alterations without consent or not to display signs without consent, if
there is a breach of that, the mischief can be removed by removing the signs or restoring the property to the
state it was in before the alterations.
I would hold that all the breaches complained of in this case were capable of remedy.
It follows that the notice under section 146 should have required them to be
remedied. As it did not, Mr Mendoza concedes, as he has to, that the notice was not a
valid one. In those circumstances there is no question of forfeiture here. '

Additional Requirements for S.146 notices


This can be found in the Leasehold Property (Repairs) Act of 1938, and applies to
all leases which were initially granted for more than 7 years and have at least 3 years
to run at the time the S.146 notice is served.
This Act ONLY APPLIES TO A TENANT IN BREACH OF A REPAIR COVENANT IN
THE LEASE.
Where this is the case, the landlord must make reference to the Leasehold Property
(Repairs) Act 1938, in the Section 146 notice so that the tenant is alerted to it. The
tenant then has 28 days to serve a counter notice, claiming the protection of the
Leasehold Property (Repairs) Act. If this happens, it ensures that the landlord cannot
proceed with forfeiture without leave of the court. Section 1(5) LPA states when
leave will be granted by a court, in essence it is where a landlord must prove there will
be a substantial reduction in the value of the freehold reversion unless the tenant
immediately remedies the breach of the repair covenant.
Once the S.146 notice is served
The landlord must allow the tenant reasonable time to carry out the remedy if the
breach is capable of remedy. If the tenant does remedy the situation and pays any
compensation due, then the lease continues as normal. If this does not happen then
the landlord can proceed to the next stage of the forfeiture procedure, which is the re-

entry stage, and is exactly the same whatever the breach of covenant.
Re-Entry
There are two types of re-entry by the landlord.
The most common type of re-entry is with a possession order from the court.
In some cases, the landlord can re-enter and take possession of the premises without
a court order which is known as a peaceable re-entry. Section 2(1) Protection from
Eviction Act 1977, applies to residential leases and requires a landlord to get a court
order if any person is lawfully residing in the premises. The landlord commits a
criminal offence if he re-enters in this way without a court order.
In Billson v Residential Apartments, the HOL determined that a landlord to use
peaceable re-entry without a court order- for commercial leases. Though they
themselves were critical of this process.
It is wise in all cases, even commercial leases, to obtain a court order, or a landlord
could face charges under
S. 6 of the Criminal Law Act 1977, which states it is a criminal offence to use or
threaten to use violence(whether to person or property) for the purposes of securing
entry into any premises where, to the knowledge of the entrant, there is someone
present who is opposed to the entry.
Relief from forfeiture
This is an important safeguard for tenants at risk of being evicted by forfeiture.
A tenant can apply to the court for relief and if granted, the landlord cannot forfeit the
lease. The lease will continue as before.
In Gill v Lewis, the court found that if the forfeiture relates to the non payment of rent,
the court has an equitable jurisdiction to grant relief, and this will always be given if
the tenant pays the rent arrears and any costs incurred by the landlord.
Relating to any other breach of covenant, then S.146(2) LPA 1925, governs the
court's power to grant relief to the tenant. This section states that a tenant can apply
for relief where a landlord is 'proceeding' to enforce a right of forfeiture. When is this?
Where a landlord is re-entering with a court possession order, then the tenant can
apply to the court for relief at any time prior to the landlord taking physical possession
of the property.
In Billson v Residential Apartments, The HOL decided that even after a landlord has
peaceably re-entered, ie without a court order, a tenant can apply to the court for
relief.

Easements
An easement is a right to make limited use of another persons land
eg, right of way, run drains under someone's land.
How to establish whether someone has or doesn't have an easement
1. Is the right capable of being an easement?, and
2. Has the right been validly acquired?
Where a right to make limited use of somebody's land fails either of these two tests it
cannot be an easement. It would more likely be a license, which is only a personal
right. Whereas a claimant will have a proprietary right.
Capacity to be an easement Re Ellenborough Park
The essential elements for an E to exist were laid out in the case of Re: Ellenborough
Park 1956, and a right will fail unless it has all of these qualities.
Additionally, the courts have held that certain rights are not acceptable as easements
even if they pass the Re: ellenborough park test. This is because they are seen to have
too adverse an impact on the other persons land.
Re : Ellenborough Park Criteria;
1. There must be a dominant and servient tenement
2. The dominant and servient tenement must not be owned and occupied
by the same person
3. The right claimed must accommodate the dominant tenement; and
4. the right claimed must be capable of being granted by deed.
1. There must be a dominant and servient tenement
For an E to exist there must be two individual pieces of land, and tenement simply
means piece of land. The one with the benefit of the E is called the dominant
tenement and the burden of the E is called the servient tenement.
2. Must be owned by different people
This is because a person cannot have an E against himself. They cannot create a right
against themselves. This works absolutely fine if both hold a different freehold, or
different leasehold estates.

3. Right claimed must 'accommodate' the dominant tenement


Accommodate here just means benefit. The essential point here is that it must be the
LAND that benefits not just the individual owner. This can be satisfied by either;

The land itself from the dominant tenement benefiting from the easement over
the servient land. Or,
The easement improving the manner in which the dominant land can be used.

Following examples of rights which have been held to accommodate the land;

A right of support from an adjoining property Directly benefiting the property


A right of way across the adjoining land. - Improving the manner in which the
land can be used
Right of drainage This could be both

An example of a purely personal benefit

A house coming with a right to watch Newcastle United play at St James' Park.
This would probably increase the value of the house, but it does not benefit the
land.

In Moody v Steggles 1879 A publican who owned a pub and the land it was on, was
given permission by a neighbour to affix a sign to his adjoining property. A court held
that this right to keep the sign on the neighbours land was capable of being an E
because it benefited the business of the pub on the land the publican owned.
So a right that benefits a land owner's business can satisfy this test to benefit his land,
as long as the business is closely associated with the dominant tenement.
This was confirmed in Wong v Beaumont Properties Ltd. A chinese restaurant owner
wanted to install a ventilation channel through the upstairs flat. As it benefited the
restaurant on the property, it was capable of being an easement.
The requirement to 'accommodate' also means that the dominant land must be close
enough to the servient tenement to be able to benefit from the easement. It does not
have to be right next to it, but an example might be an owner given permission to
park within walking distance.
4. The easement must be capable of being granted by deed.
This does not require a deed to have actually been used to create the E.
It requires that it could only be an E if it could have been written up in the form of a
deed.
Case law has given two consequences;
1. The right must be sufficiently definite it must be 'capable of precise definition

and description'.
This had led the courts to hold that neither a claim to a good view across a
neighbouring land nor a right to receive a good tv signal across neighbouring
land could succeed as easement's as they were too vague.
2. The parties to the easement must have the capacity to grant and receive it
This means they should both have an estate in the land because an E is a
proprietary interest. Thus a licensee would lack capacity to grant or receive an E
because a license is not an estate in land.
IT DOES NOT MEAN A DEED HAS ACTUALLY BEEN USED!!!
Claims which are not easements even if they satisfy RE: ELLENBOROUGH
PARK

The first of these is a claim which would require the servient owner to take a positive
action. It is an essential idea for easements that the servient owner shouldn't have to
do anything and in particular, shouldn't have to spend any money.
In Regis Property Co Ltd v Redman The right at stake was a claim to have an E to
receive a constant supply of hot water from an adjoining property. The COA held this
could not be an E as it would involve the servient owner of the adjoining property in
both cost and effort.
There is a famous exception to this rule and this is the right to have a dividing fence
maintained. The COA in Crow v Wood, held that in limited circumstances a right to
have a dividing fence maintained could be an easement, even though the servient
owner would be involved in cost and action in taking care of the fence.
An area the courts have wrestled with is whether a claim that left the servient owner
without reasonable use of their own land could be acceptable as an E. This issue has
arisen in relation to easements of storage and more recently, car parking.
An easement of storage is simply the right to store something on someone else's land,
such as coal in a neighbour's coal shed. In principle this could be, as it would be no
problem if it just took up a small corner of the shed, however what if it filled it up?
Until 2007 it was quite clear that an easement of storage would fail if it left the
servient owner without reasonable use of the servient land. This was said in the case
of London & Blenheim estates v Ladbroke.
Batchelor v Marlow 2001, In this case Batchelor owned a strip of land and Marlow who
owned an adjoining strip of land claimed an easement to park up to 6 cars on the
verge of the road (owned by Batchelor) between 8:30 6:00 Monday to Friday. The
COA this is not an easement, It said that such a restriction would make Batchelor's
ownership of that land little more than an illusion.
Similarly, the idea that the owner must not be deprived of use has led to the idea that
there cannot be an easement to park in a specific car- sized space, though there could
be a right to park in an unspecified spot within a large area.
However, in a Scottish case, Moncrieff v Jamieson, Lord Neuberger stated that a right
could be an easement even if the servient owner effectively cannot use the property.

What matters is that they retain possession and control.


As this was a Scottish case it is only persuasive, and in the 2008 case of Virdi v Chana,
the judge noted the comments in Moncrieff, the court felt bound to follow the decision
by the COA in Batchelor v Marlow, as it remains good law in England. So the test for
the time being remains the 'reasonable use' test set out in London & Blenheim Estates
v Ladbroke.
These are the requirements for whether something is capable of being an easement
using the rules in RE: Ellenborough Park. Then we discussed that even some rights
that satisfy these requirements cannot be easements if they require a positive action
from the servient owner. If any of these rights are not met, it will not be an E, it would
likely only be a license.

Has it been validly acquired?


Assuming a claim satisfies the test of capacity, we must move on to consider if it has
been validly acquired by the claimant.
A GRANT of an easement occurs when a landowner GIVES an E over their own land to
another estate owner. An owner giving someone a right over the land they own to
another estate owner.
A RESERVATION is the reverse situation. An owner keeping a right over land they have
sold off.
Grant
A grant can occur either expressly or impliedly.
An express grant is one that is made by actual written agreement between the parties.
This agreement could happen upon sale, or it could happen where the two plots of
land are separately owned before the E is granted.
An implied grant arises where, in certain circumstances, the law will imply that a buyer
has obtained an E even though there has been nothing expressly agreed between the
parties. There are 4 occasions where an implied grant might arise;
1.
2.
3.
4.

When an E is implied out of necessity


When the court can infer there was an E of common intended use.
What is known as the rule in Wheeldon v Burrows
By virtue of S.62 LPA 1925

Acquisition of an easement by implied reservation Necessity


As in the case of a grant, an easement can be reserved expressly, or the law may
imply an easement.
An E of necessity is one which without the Dominant land cannot be used at all. It is
not sufficient that it is convenient. It is not even sufficient that it is reasonably
necessary for its use. It must be absolutely necessary. Without it the owner would not
be able to derive any benefit from his land.

The classic illustration of this is where the land is landlocked. If you buy a plot of land
and find that your sole means of access to it is over the seller's retained land, then in
that case you may be able to claim an implied easement of necessity over the seller's
land.
The basis for implying an easement of necessity. In Nickerson v Barraclough,i the COA
stated that if the servient owner can show no easement was intended, the dominant
owner will not get the easement even if it means they cannot use the land at all. This
would happen if the deed expressly says that no right of access is being given, the
buyer would be left having to sue the solicitor.
However, in the case of an easement obtained by reservation only two methods are
available; necessity and common intention. Thus NEITHER Wheeldon v Burrows, or
S.62 LPA 1925 apply to reservation. The reason for this is the courts prefer implied
grants to implied reservations. The judicial view is that the seller knows the land and
has control of the situation, therefore when they sell their land they ought to consider
in advance what easements they will need over it and should expressly reserve them
at the time of sale.
Sweet v Sommer 2005 A right of way for vehicles might be impliedly created as an
easement of necessity.

An E of common intended use is an E that will come into existence if both parties had
intended that the dominant land be used for a particular purpose and that purpose can
only be achieved by granting the easement.
Stafford v Lee, - Where a seller sold off a plot of land and on the plan of the plot it
showed as having a house on it, although at the time of sale there was only woodland.
When a later purchaser of the plot got planning permission to build a house on it, he
found he would only be able to by bringing construction materials over the seller's
retained land. The COA held the purchaser's claim as the plan of the plot showed a
common intention that a house should be built on the plot. Since the means to achieve
this was to bring materials over the seller's land an easement of common intention
could be implied.
Common intention easements are another form of necessity. The easement claimed
must be the ONLY way in which the parties common intention could have been
achieved. If there is another way, it will fail.
The overlap is demonstrated by Wong v Beaumont Property Trust Ltd, where the
owner of the restaurant was required to install a suitable ventilation system through
the upstairs flat which was retained by the company. The COA took the view that an
easement would be implied and decided this on the basis of necessity as the terms in
the lease didn't allow it to be used for anything other than a restaurant. However, the
facts could have been easily construed as an easement of common intended use,
since without the ventilation the property could not have been used for its intended
purpose as a restaurant.
Acquisition of an easement by implied grant Wheeldon v Burrows & S.62
LPA 1925

Wheeldon v Burrows
An owner exercises a right over one part of his land for the benefit of another part of
his land. Eg, a part of land with a house on it, and has another plot of land which he
runs drainage pipes underneath for the benefit of the house.
The owner sells the part of land which had the benefit of the right and the owner
retains that part over which the right was exercised. Eg, he sold the house, but
retained the plot of land with the drainage pipes running through it.
In this situation the buyer may acquire an implied easement to continue using that
same right over the owner's retained land.
3 conditions must be met before the easement will be impliedly granted;
1. The right claimed must have been in use by the OWNER at the time of sale, and
2. It must have been continuous or apparent at the time of sale, and (maybe or,)
3. It must be necessary to the reasonable enjoyment of the land sold.
1. The right claimed must have been in use by the owner at the time of sale
It must be the owner, it is not enough that it is someone else who is using the right.
While the owner is using the right, it is considered a 'quasi' easement, this means it
has the ability to turn into an easement. You cannot have an easement against
yourself.
2. It must have been continuous or apparent at the time of sale
Continuous does not mean constant use. It must have been regularly used by the
owner before sale.
As well as being continuous it must have been apparent. The word apparent carries
the idea that evidence of the right must be visible, there must be some evidence of
existence of the easement on the land. Eg, a quasi easement of a right to drainage
might be evidenced by the existence of pipes crossing the servient land.
A quasi easement is apparent if there is a permanent feature on the servient land
which signals the existence of an easement Ward v Kirkland In this case there was
nothing apparent to indicate it's existence.
3. The right must be necessary for the reasonable enjoyment of the land
This isn't the same as necessity, you don't have to show the land couldn't be used at
all without the easement. However, it isn't merely convenience either, it must be
necessary to the reasonable enjoyment of the land. Millman v Ellis The claimant
bought a house from ellis, to get to the house he needed to use a layby which ran over
Ellis' land, he was granted an easement over part of the lay by, but it was extremely
dangerous unless he could use the whole of it. The court upheld his claim as it was
necessary for the reasonable enjoyment.
Contrast this to Wheeler v JJ Saunders Ltd, which failed. The claimants bought
farmhouse with 2 possible entrances, the 2nd of which involved crossing the seller's
retained land. A majority in the COA held that the first entrance would do just as well
and mere convenience wasn't sufficient.

There is some debate as to whether both 2 and 3 need to be satisfied or just one of
them. In Millman v Ellis, it was held they BOTH need to be satisfied. In any event, the
two requirements overlap and interact so if one is present the other one usually is too.

S.62 LPA 1925


A S.62 Easement is sometimes called an express easement in the textbooks, but in
this syllabus it is treated as an implied easement.
S.62 states;
'A conveyance of land shall be deemed to include and shall operate to convey with the
land, all liberties, privileges, easements, rights and advantages whatsoever,
appertaining to the land, or at the time of conveyance, enjoyed with the land.'
What the section is doing is setting out all the benefits that will pass to a buyer when
the legal title to land is conveyed. It refers to all existing advantages pass to the
buyer. The courts have held that which was before the transfer simply personal, an
advantage, will pass to the buyer as a property right, an easement.
1. Part of an owners land is occupied by someone else.
2. The occupier is allowed, by the owner, to make some use of the owner's
retained land. This is not a property right, only a personal license at this stage.
3. At some point after the permission was given, there is then a conveyance by
deed of a legal estate in the occupied plot.
When the conveyance takes place, S.62 converts the personal permission into an
easement.
What is required for S.62 to take effect?
1. There must be a conveyance This is a document which creates or transfers a
legal estate.
Eg; transfer of legal freehold/leasehold by deed, grant of a legal lease by deed,
renewal of a legal lease by deed. Notice that a contract to transfer or lease land
would not be sufficient as it does not give rise to a legal estate.
2. The advantage claimed must have been 'enjoyed with' the land that is being
sold. In Wood v Waddington, it was held that it must be evident before the
conveyance that one part owner's land carries a benefit which arises from a
burden on the other part. In other words there needs to be an identifiable a
quasi dominant and servient tenement before sale.
Diversity of occupation as a means of showing that the right claimed was
enjoyed with the land sold.
One obvious way to show the benefit/burden relationship is to have the benefited part
occupied by someone other than the owner and using the advantage over the
burdened part occupied by the owner before the two parts are separated via sale.
There is a line of authority that S.62 requires this diversity of occupation, notably
Long v Gowlett, and supported obiter in Sovmots Investments v Secretary of State for

the Environment 1979.


Wright v Macadam- Mr Macadam leased part of his land to Mrs Wright. Separately to
the lease, he gave her a license to use the shed for storing coal. When the lease ran
out, he then granted her a new legal lease to Mrs Wright. The grant of this new lease
would have amounted to a conveyance by deed of a legal estate so it converted the
license to use the shed into an easement.
Where diversity of occupation is helping prove the advantage is enjoyed with the land
note two points;
1. The diversity must continue right up to the time of the conveyance. Payne v
Inwood demonstrates that even one week common occupation of both
tenements by the same person immediately before the conveyance will break
the diversity of occupation rule and prevent S.62 from taking effect.
2. It does not matter on what basis the land is being occupied, ie whether its as a
licensee or a tenant.
Rights being continuous and apparent as an alternative means of showing
the right claimed was enjoyed with the land sold.
This is an alternative to diversity of occupation, and has been approved in cases from
P and S Platt and Wood v Waddington 2015.
These cases confirm that even where there has been no diversity of occupation before
the sale, S.62 can still operate if the seller himself was exercising the right claimed
over part of his land for the benefit of another and that right was continuous and
apparent.
Continuous and apparent carry the same meaning that they do in Wheeldon v
Burrows, which Alford v Hannaford provides confirmation for. Therefore continuous
means that the quasi right must have been used regularly by the owner before the
sale. The term apparent means that before the sale, there must be some visible signs
on the servient land of the existence of the right claimed.
Therefore a quasi easement that is continuous and apparent can highlight that part of
the land that is dominant and the part that is servient quite as well as diversity of
occupation.
Thus a buyer can claim to have acquired an easement under S.62 LPA 1925, either if
the land was transferred to them by a conveyance and the right claimed had been
used, with the owner's permission, by someone other than the owner before sale, or
the owner himself had exercised that right claimed and it was both continuous and
apparent.
Comparison
1. S.62 will operate where there was a diversity of occupation before sale,
Wheeldon v Burrows will not.
2. Both S.62 and Wheeldon v Burrows will operate where the owner was in
occupation before sale. Where this is the case, both methods require the right
claimed to be both continuous and apparent. In addition Wheeldon requires the
right to be necessary to the reasonable enjoyment of the land, but this is not
needed in S.62.

Excluding Wheeldon v Burrows and S.62


A seller can prevent both methods from happening by expressly stating in the deed of
sale that they are not to apply. In practice this is commonly done to stop the buyer
acquiring easements by these methods.

Transfer of an existing easement on sale of the dominant or servient


tenement

Sale of dominant tenement


This applies no matter how the easement is created, by grant or reservation, expressly
or impliedly.
The benefit of an existing easement will transfer. It can be transferred expressly when
the dominant land is sold. Alternatively, if there was no express transfer, S.62 LPA
1925 will impliedly transfer the benefit of an existing easement on sale. So S.62 has
two functions, firstly it can operate to impliedly grant an easement and secondly, it
will operate to pass on the benefit of a pre-existing easement.
Sale of servient tenement
The first point to remember is that in registered land, all rights, including easements
will automatically bind a transferee of land unless that transferee is a purchaser for
valuable consideration who has registered their ownership. If we are dealing with such
a purchaser, then they enjoy protected status under S.29 LRA 2002. And S.29 then
states they will not be bound by any interest unless it has either been recorded on the
charges register by way of notice or if it is an overriding interest.
In the case of express easements, the easement will always need to be recorded on
the charges register of the servient title for the new owner to be bound. This is true
whether the easement is legal or equitable.
Express easements are not overriding.
What about implied easements? An implied easement can too go onto the charges
register to bind the purchaser. If this didn't happen, implied easements can be an
overriding interest under Schedule 3 Paragraph 3, it will override if it satisfied any one
of those criteria.
1. That they had actual knowledge of the easement
2. If it would have been obvious upon reasonable careful inspection of the land, or

3. If it had been used up to one year leading up to the purchaser buying the land.

You might also like